Question

QUESTION 17 Which of the following statements is most correct? a. An increase in expected inflation...

QUESTION 17

  1. Which of the following statements is most correct?

    a. An increase in expected inflation could be expected to increase the required return on a riskless asset and on an average stock by the same amount, other things held constant.

    b. A graph of the SML would show required rates of return on the vertical axis and standard deviations of returns on the horizontal axis.

    c. If two "normal" or "typical" stocks were combined to form a 2-stock portfolio, the portfolio's expected return would be a weighted average of the stocks' expected returns, but the portfolio's standard deviation would probably be greater than the average of the stocks' standard deviations.

    d. If investors became more averse to risk, then (1) the slope of the SML would increase and (2) the required rate of return on low-beta stocks would increase by more than the required return on high-beta stocks.

    e. The CAPM has been thoroughly tested, and the theory has been confirmed beyond any reasonable doubt.

Homework Answers

Answer #1

When there would be an increase in the expected inflation, that would be expected to increase the required return on a riskless asset because the increase in expected return of riskless asset and average stock by will be of same amount.

when there would be constant risk free rate and constant market risk premium then the increase in inflation will be leading to increase in expected return by same amount of both the risk free Assets and average assets.

other statements are not correct because they are not representing the correct scenarios.

Correct answer would be option (A).

Know the answer?
Your Answer:

Post as a guest

Your Name:

What's your source?

Earn Coins

Coins can be redeemed for fabulous gifts.

Not the answer you're looking for?
Ask your own homework help question
Similar Questions
Suppose that the CAPM is the correct model of expected returns. Which of the following stocks...
Suppose that the CAPM is the correct model of expected returns. Which of the following stocks will have the highest expected return? Group of answer choices A stock with beta of 0.8 and a standard deviation of returns equal to 20% A stock with beta of 0.65 and a standard deviation of returns equal to 27.5% A stock with beta of 0.5 and a standard deviation of returns equal to 35% A stock with beta of 0.6 and a standard...
Q1. Which of the following statements about the portfolio is true? a. The expected return of...
Q1. Which of the following statements about the portfolio is true? a. The expected return of a portfolio is NOT the weighted average of the expected returns of all individual stocks in the portfolio. b. The standard deviation of a portfolio is NOT the weighted average of the standard deviations of all individual stocks in the portfolio. c. Portfolio beta is NOT the weighted average of the beta values of all individual stocks in the portfolio Q2. Which of the...
Which of the following statements is CORRECT? Select one: a. The beta of a portfolio of...
Which of the following statements is CORRECT? Select one: a. The beta of a portfolio of stocks is always smaller than the betas of any of the individual stocks. b. The beta of a portfolio of stocks is always larger than the betas of any of the individual stocks. c. It is theoretically possible for a stock to have a beta of 1.0. If a stock did have a beta of 1.0, then, at least in theory, its required rate...
Which of the following Statements is most accurate? The greater the number of stocks in a...
Which of the following Statements is most accurate? The greater the number of stocks in a stock portfolio, the harder it would be to outperform a market index like the S&P 500 The greater the number of stocks in a stock portfolio, the higher the beta of the portfolio The greater the number of stocks in a stock portfolio, the lower the portfolio's systematic risk The beta of a sock portfolio will increase any time that one or more of...
Which of the following statements is CORRECT? Select one: a. Collections Inc. is in the business...
Which of the following statements is CORRECT? Select one: a. Collections Inc. is in the business of collecting past-due accounts for other companies, i.e., it is a collection agency. Collections' revenues, profits, and stock price tend to rise during recessions. This suggests that Collections Inc.'s beta should be quite high, say 2.0, because it does so much better than most other companies when the economy is weak. b. Suppose the returns on two stocks are negatively correlated. One has a...
Which of the following statements is/are incorrect? 1) A security's beta measures its market risk. 2)...
Which of the following statements is/are incorrect? 1) A security's beta measures its market risk. 2) If investors become less risk averse, the slope of SML will decrease accordingly. 3) The tighter the probability distribution of its expected future return, the greater risk of a given investment as measured by its standard deviation. 4) SML is a graphical depiction of CAPM?
Which of the following will be true about the return and standard deviation of a portfolio?...
Which of the following will be true about the return and standard deviation of a portfolio? A. The return of a portfolio will be the weighted average of the returns in the portfolio, but the standard deviation will be less than the weighted average of the standard deviations in the portfolio. B. The return and standard deviation of a portfolio will be the weighted average of the returns and standard deviations in the portfolio. C. The return and standard deviation...
Which two of the following five statements are correct? Select two alternatives: A risk-averse investor will...
Which two of the following five statements are correct? Select two alternatives: A risk-averse investor will avoid investing in stocks. Diversification eliminates systematic risk but not idiosyncratic risk. The 95% confidence interval for the expected return is defined as the Historical Average Return plus or minus three standard errors. The realized return is the total return we earn from dividends and capital gains, expressed as a percentage of the initial stock price. While there is no clear relationship between risk...
Stock X has a 10% expected return, a beta coefficient of 0.9, and a 35% standard...
Stock X has a 10% expected return, a beta coefficient of 0.9, and a 35% standard deviation of expected returns. Stock Y has a 12.5% expected return, a beta coefficient of 1.2, and a 25% standard deviation. The risk-free rate is 6%, and the market risk premium is 5%. a. Calculate each stock’s coefficient of variation. b. Which stock is riskier for a diversified investor? c. Calculate each stock’s required rate of return. d. On the basis of the two...
Stock X has a 10% expected return, a beta coefficient of 0.9, and a 35% standard...
Stock X has a 10% expected return, a beta coefficient of 0.9, and a 35% standard deviation of expected returns. Stock Y has a 12.5% expected return, a beta coefficient of 1.2, and a 25% standard deviation. The risk-free rate is 6%, and the market risk premium is 5%. a. Calculate each stock’s coefficient of variation. b. Which stock is riskier for a diversified investor? c. Calculate each stock’s required rate of return. d. On the basis of the two...
ADVERTISEMENT
Need Online Homework Help?

Get Answers For Free
Most questions answered within 1 hours.

Ask a Question
ADVERTISEMENT